The goal of this set of notes is to improve our problem solving and proof writing skills. You are encouraged to submit a solution to any of these problems, and join in the discussion in #imo-discussion on Saturday 7 at 9:00 pm IST ,8 30 PDT. For more details, see IMO Problems Discussion Group.
Hello Problem Solvers, I have lately been absolutely busy with the academics. Really sorry for the weekly delay! Here are the problems from the 3rd IMO. I recommend everyone to try them and post any progress they make!
Q1. (HUN) Solve the following system of equations: where and are given real numbers. What conditions must hold on and for the solutions to be positive and distinct?
Q2. (POL) Let , and be the lengths of a triangle whose area is . Prove that . In what case does equality hold?
Q3. (BUL) Solve the equation , where is a given positive integer.
Easy Math Editor
This discussion board is a place to discuss our Daily Challenges and the math and science related to those challenges. Explanations are more than just a solution — they should explain the steps and thinking strategies that you used to obtain the solution. Comments should further the discussion of math and science.
When posting on Brilliant:
*italics*
or_italics_
**bold**
or__bold__
paragraph 1
paragraph 2
[example link](https://brilliant.org)
> This is a quote
\(
...\)
or\[
...\]
to ensure proper formatting.2 \times 3
2^{34}
a_{i-1}
\frac{2}{3}
\sqrt{2}
\sum_{i=1}^3
\sin \theta
\boxed{123}
Comments
2) Just use Heron's Formula:
Replace S with s(s−a)(s−b)(s−c) where s=2a+b+c
We now have
4316(a+b+c)(a+b−c)(a−b+c)(−a+b+c)=3−(a4+b4+c4−2a2b2−2b2c2−c2a2)
Squaring both sides of the original inequality, we have
a4+b4+c4+2a2b2+2b2c2+2c2a2≥−3a4−3b4−3c4+6a2b2+6b2c2+6c2a2
4a4+4b4+4c4≥4a2b2+4b2c2+4c2a2
a4+b4+c4≥a2b2+b2c2+c2a2
Since a2b2≤2a4+b4 by AM-GM, we have the above inequality is true with equality at a=b=c, which is an equilateral triangle.
Log in to reply
One of the great many proofs the problem has! Nice short and effective
My solution to 1 (Probably the long way around):
We have x2+y2=(x+y)2−2xy=(x+y)2−2z2. Putting this in, we now have
(x+y)+z=a
(x+y)2−z2=b2
Note that ((x+y)+z)((x+y)−z)=(x+y)2−z2. We can now split this into 2 cases: when a=0 and when a=0.
When a=0, we have b=0 and x+y=−z, which gives us xy=(x+y)2. Then, we get x2+xy+y2=0. Multiplying by (x−y), we have x3−y3=0 which means x=y.This means that x2+x2+x2=0, which gives x=0. Thus, there are no positive solutions in this case.
When a=0, we have
(x+y)+z=a
(x+y)−z=ab2
This has solutions (x+y)=2aa2+b2 and z=2aa2−b2. Thus a2>b2. Now, we have (x+y)=2aa2+b2 and xy=(2aa2−b2)2. This means that x and y are roots of the quadratic k2−2aa2+b2k+(2aa2−b2)2. The discriminant of this quadratic is
(2aa2+b2)2−4(2aa2−b2)2=4a2(3a2−b2)(3b2−a2)
This final value must be positive, which means that either both 3a2−b2 and 3b2−a2 are negative or both are positive. If they are both negative, 2(a2+b2) is negative, which is incorrect. Therefore 3a2>b2 and 3b2>a2. But we have already replaced the first inequality with the sharper bound a2>b2. It is clear that both roots of the quadratic must be positive if the discriminant is positive (we can see this either from (2aa2+b2)2>(2aa2+b2)2−(22aa2−b2)2). We have now found the solutions to the system, and determined that it has positive solutions if and only if a is positive and 3b2>a2>b2.
Log in to reply
The solution is correct (if not short!) Great work @Sharky Kesa
Log in to reply
I posted solution to Q2. Need to sleep now. Let me think about 3 later.
@Sharky Kesa @Abdur Rehman Zahid @Xuming Liang This time its Algebra!
3) My solution - hopefully not too short
We have that cos2x+sin2x=1 so we can't have solutions with n=2 and 0<∣cos(x)∣,∣sin(x)∣<1. Also, we can't have solutions when n=2 since the sign is wrong (needs to be '-', not '+'). So the only solutions have cosx=0 or sinx=0, and these are:
x=kπ for any integral k and n is even,
x=2k(π) for any integral k and n is odd,
x=2k(π+23π) for any integral k and n is odd.
Log in to reply
@sharky An integral part of the solution is still missing, I.e. Why n=2 doesnt have solutions. Though you created the bound, complete your proof by adding that in!